What if Force was not Mass times Acceleration?2019 Community Moderator Election Unicorn Meta Zoo #1: Why another podcast? Announcing the arrival of Valued Associate #679: Cesar Manara 2019 Moderator Election Q&A - QuestionnaireHow do we know that $F = ma$, not $F = k cdot ma$Why isn't it $E approx 27.642 times mc^2$?Physical meaning of force times areaHow did Newton find out force has something to do with acceleration?Mass, Acceleration, and ForceCalculating mass of an orbiting body with force and accelerationAcceleration due to gravitational forceHow force is mass times acceleration?Newton's second law with $F=kma$Force without accelerationHow force is equal to the product of mass and instantaneous acceleration?Force and center of mass acceleration

"My boss was furious with me and I have been fired" vs. "My boss was furious with me and I was fired"

Password Generator in batch

What to do with someone that cheated their way through university and a PhD program?

Arriving in Atlanta after US Preclearance in Dublin. Will I go through TSA security in Atlanta to transfer to a connecting flight?

Will I lose my paid in full property

Could moose/elk survive in the Amazon forest?

Visa-free travel to the US using refugee travel document from Spain?

What *exactly* is electrical current, voltage, and resistance?

My bank got bought out, am I now going to have to start filing tax returns in a different state?

What is it called when you ride around on your front wheel?

What is this word supposed to be?

My admission is revoked after accepting the admission offer

Error: Syntax error. Missing ')' for CASE Statement

Why did Israel vote against lifting the American embargo on Cuba?

Multiple options vs single option UI

What is "leading note" and what does it mean to "raise a note"?

Align column where each cell has two decimals with siunitx

Is there any hidden 'W' sound after 'comment' in : Comment est-elle?

The art of proof summarizing. Are there known rules, or is it a purely common sense matter?

I preordered a game on my Xbox while on the home screen of my friend's account. Which of us owns the game?

Mistake in years of experience in resume?

Co-worker works way more than he should

Putting Ant-Man on house arrest

What is the ongoing value of the Kanban board to the developers as opposed to management



What if Force was not Mass times Acceleration?



2019 Community Moderator Election
Unicorn Meta Zoo #1: Why another podcast?
Announcing the arrival of Valued Associate #679: Cesar Manara
2019 Moderator Election Q&A - QuestionnaireHow do we know that $F = ma$, not $F = k cdot ma$Why isn't it $E approx 27.642 times mc^2$?Physical meaning of force times areaHow did Newton find out force has something to do with acceleration?Mass, Acceleration, and ForceCalculating mass of an orbiting body with force and accelerationAcceleration due to gravitational forceHow force is mass times acceleration?Newton's second law with $F=kma$Force without accelerationHow force is equal to the product of mass and instantaneous acceleration?Force and center of mass acceleration










2












$begingroup$


We know that Force equals Mass times Acceleration, $F = ma$. But what if we had a law like $$F=m^2a$$
or
$$F = 2ma~?$$










share|cite|improve this question









New contributor




pooja paliwal is a new contributor to this site. Take care in asking for clarification, commenting, and answering.
Check out our Code of Conduct.







$endgroup$







  • 8




    $begingroup$
    As we all know we live in a classical world where everything works on the basis on this law Ever heard of quantum mechanics?
    $endgroup$
    – Gert
    9 hours ago






  • 1




    $begingroup$
    Related, if not dupe, physics.stackexchange.com/q/104101/25301
    $endgroup$
    – Kyle Kanos
    9 hours ago






  • 3




    $begingroup$
    The proposed duplicate asks why there's no numerical constant out front (see also), but the question of why the constant of proportionality is "mass" rather than "mass squared" (or the like) is an interesting twist.
    $endgroup$
    – rob
    9 hours ago






  • 4




    $begingroup$
    Ultimately, this depends on what your particular definition of mass is. If by "mass" you mean "inertial mass," then the question is trivial - inertial mass is defined to be the proportionality constant between force and acceleration. So which definition of mass are we intended to use?
    $endgroup$
    – probably_someone
    9 hours ago










  • $begingroup$
    Without a clear definition of the terms mass and force the question is too broad for a reasonably confined answer.
    $endgroup$
    – GiorgioP
    7 hours ago















2












$begingroup$


We know that Force equals Mass times Acceleration, $F = ma$. But what if we had a law like $$F=m^2a$$
or
$$F = 2ma~?$$










share|cite|improve this question









New contributor




pooja paliwal is a new contributor to this site. Take care in asking for clarification, commenting, and answering.
Check out our Code of Conduct.







$endgroup$







  • 8




    $begingroup$
    As we all know we live in a classical world where everything works on the basis on this law Ever heard of quantum mechanics?
    $endgroup$
    – Gert
    9 hours ago






  • 1




    $begingroup$
    Related, if not dupe, physics.stackexchange.com/q/104101/25301
    $endgroup$
    – Kyle Kanos
    9 hours ago






  • 3




    $begingroup$
    The proposed duplicate asks why there's no numerical constant out front (see also), but the question of why the constant of proportionality is "mass" rather than "mass squared" (or the like) is an interesting twist.
    $endgroup$
    – rob
    9 hours ago






  • 4




    $begingroup$
    Ultimately, this depends on what your particular definition of mass is. If by "mass" you mean "inertial mass," then the question is trivial - inertial mass is defined to be the proportionality constant between force and acceleration. So which definition of mass are we intended to use?
    $endgroup$
    – probably_someone
    9 hours ago










  • $begingroup$
    Without a clear definition of the terms mass and force the question is too broad for a reasonably confined answer.
    $endgroup$
    – GiorgioP
    7 hours ago













2












2








2





$begingroup$


We know that Force equals Mass times Acceleration, $F = ma$. But what if we had a law like $$F=m^2a$$
or
$$F = 2ma~?$$










share|cite|improve this question









New contributor




pooja paliwal is a new contributor to this site. Take care in asking for clarification, commenting, and answering.
Check out our Code of Conduct.







$endgroup$




We know that Force equals Mass times Acceleration, $F = ma$. But what if we had a law like $$F=m^2a$$
or
$$F = 2ma~?$$







newtonian-mechanics forces mass acceleration inertia






share|cite|improve this question









New contributor




pooja paliwal is a new contributor to this site. Take care in asking for clarification, commenting, and answering.
Check out our Code of Conduct.











share|cite|improve this question









New contributor




pooja paliwal is a new contributor to this site. Take care in asking for clarification, commenting, and answering.
Check out our Code of Conduct.









share|cite|improve this question




share|cite|improve this question








edited 5 hours ago









Qmechanic

108k122011253




108k122011253






New contributor




pooja paliwal is a new contributor to this site. Take care in asking for clarification, commenting, and answering.
Check out our Code of Conduct.









asked 9 hours ago









pooja paliwalpooja paliwal

194




194




New contributor




pooja paliwal is a new contributor to this site. Take care in asking for clarification, commenting, and answering.
Check out our Code of Conduct.





New contributor





pooja paliwal is a new contributor to this site. Take care in asking for clarification, commenting, and answering.
Check out our Code of Conduct.






pooja paliwal is a new contributor to this site. Take care in asking for clarification, commenting, and answering.
Check out our Code of Conduct.







  • 8




    $begingroup$
    As we all know we live in a classical world where everything works on the basis on this law Ever heard of quantum mechanics?
    $endgroup$
    – Gert
    9 hours ago






  • 1




    $begingroup$
    Related, if not dupe, physics.stackexchange.com/q/104101/25301
    $endgroup$
    – Kyle Kanos
    9 hours ago






  • 3




    $begingroup$
    The proposed duplicate asks why there's no numerical constant out front (see also), but the question of why the constant of proportionality is "mass" rather than "mass squared" (or the like) is an interesting twist.
    $endgroup$
    – rob
    9 hours ago






  • 4




    $begingroup$
    Ultimately, this depends on what your particular definition of mass is. If by "mass" you mean "inertial mass," then the question is trivial - inertial mass is defined to be the proportionality constant between force and acceleration. So which definition of mass are we intended to use?
    $endgroup$
    – probably_someone
    9 hours ago










  • $begingroup$
    Without a clear definition of the terms mass and force the question is too broad for a reasonably confined answer.
    $endgroup$
    – GiorgioP
    7 hours ago












  • 8




    $begingroup$
    As we all know we live in a classical world where everything works on the basis on this law Ever heard of quantum mechanics?
    $endgroup$
    – Gert
    9 hours ago






  • 1




    $begingroup$
    Related, if not dupe, physics.stackexchange.com/q/104101/25301
    $endgroup$
    – Kyle Kanos
    9 hours ago






  • 3




    $begingroup$
    The proposed duplicate asks why there's no numerical constant out front (see also), but the question of why the constant of proportionality is "mass" rather than "mass squared" (or the like) is an interesting twist.
    $endgroup$
    – rob
    9 hours ago






  • 4




    $begingroup$
    Ultimately, this depends on what your particular definition of mass is. If by "mass" you mean "inertial mass," then the question is trivial - inertial mass is defined to be the proportionality constant between force and acceleration. So which definition of mass are we intended to use?
    $endgroup$
    – probably_someone
    9 hours ago










  • $begingroup$
    Without a clear definition of the terms mass and force the question is too broad for a reasonably confined answer.
    $endgroup$
    – GiorgioP
    7 hours ago







8




8




$begingroup$
As we all know we live in a classical world where everything works on the basis on this law Ever heard of quantum mechanics?
$endgroup$
– Gert
9 hours ago




$begingroup$
As we all know we live in a classical world where everything works on the basis on this law Ever heard of quantum mechanics?
$endgroup$
– Gert
9 hours ago




1




1




$begingroup$
Related, if not dupe, physics.stackexchange.com/q/104101/25301
$endgroup$
– Kyle Kanos
9 hours ago




$begingroup$
Related, if not dupe, physics.stackexchange.com/q/104101/25301
$endgroup$
– Kyle Kanos
9 hours ago




3




3




$begingroup$
The proposed duplicate asks why there's no numerical constant out front (see also), but the question of why the constant of proportionality is "mass" rather than "mass squared" (or the like) is an interesting twist.
$endgroup$
– rob
9 hours ago




$begingroup$
The proposed duplicate asks why there's no numerical constant out front (see also), but the question of why the constant of proportionality is "mass" rather than "mass squared" (or the like) is an interesting twist.
$endgroup$
– rob
9 hours ago




4




4




$begingroup$
Ultimately, this depends on what your particular definition of mass is. If by "mass" you mean "inertial mass," then the question is trivial - inertial mass is defined to be the proportionality constant between force and acceleration. So which definition of mass are we intended to use?
$endgroup$
– probably_someone
9 hours ago




$begingroup$
Ultimately, this depends on what your particular definition of mass is. If by "mass" you mean "inertial mass," then the question is trivial - inertial mass is defined to be the proportionality constant between force and acceleration. So which definition of mass are we intended to use?
$endgroup$
– probably_someone
9 hours ago












$begingroup$
Without a clear definition of the terms mass and force the question is too broad for a reasonably confined answer.
$endgroup$
– GiorgioP
7 hours ago




$begingroup$
Without a clear definition of the terms mass and force the question is too broad for a reasonably confined answer.
$endgroup$
– GiorgioP
7 hours ago










9 Answers
9






active

oldest

votes


















2












$begingroup$

It wouldn't change anything. We would just have defined mass differently. What is relevant about Newton's 2nd law is not $m$ but rather the fact that $F$ and $a$ are proportional, $Fpropto a$.



The proportionality constant can then be called $m$ or $2m$ or $m^2$. That doesn't matter for the relationship that this law describes.



Newton's 2nd law defines mass (inertial mass) as the proportionality constant in $F=ma$. Had we decided upon $F=2ma$, then we would just have defined mass as half of the proportionality constant. It would change our language regarding mass (you would only weigh 40 kg rather than 80 kg i.e.), but it wouldn't change the way the world works.



If you on the other hand had asked about $F=ma^2$ or so, then you are messing with the world entirely with disastrous consequences that we can only try to guess as a result!






share|cite|improve this answer









$endgroup$








  • 6




    $begingroup$
    It would change something though as we wouldn’t have eg universal acceleration. Galileo would not have found the balls hit the ground at the same time when dropped from the tower ;)
    $endgroup$
    – innisfree
    8 hours ago






  • 8




    $begingroup$
    I disagree that it would be so simple. It would also have the consequence that when putting two objects together, we would have to add their masses squared instead of their masses, which seems pretty strange. Clearly what we now call mass is a better option than the others.
    $endgroup$
    – Javier
    8 hours ago






  • 1




    $begingroup$
    We’d also have to use some different units or fix the units somehow, eg introduce another dimensionful constant of nature or maybe just change the units of Newton’s constant $G$
    $endgroup$
    – innisfree
    8 hours ago






  • 3




    $begingroup$
    It would be nice if this answer (which is the accepted one) made mention of gravitational mass (and other non-inertial properties we expect mass to have, like additivity) and how they change.
    $endgroup$
    – jacob1729
    8 hours ago






  • 1




    $begingroup$
    If it was $m^2$ then it would definitely change something if the $m$ also refers to the mass associated with the force of gravity
    $endgroup$
    – Aaron Stevens
    5 hours ago



















6












$begingroup$

In contrast to the other answers, I am going to go a different direction. This is not a question of mathematics, where we can just say "oh yeah just redefine your variables." This is a physics question.



Going further, The OP is assuming $m$ stands for the mass of an object, not some arbitrary proportionality constant that ends up being the mass. Therefore, having a law like $F=km^2a$ ($k$ as just some constant for unit's sake) would not leave the universe unchanged. This is saying, for example, that if we double the "amount of stuff" associated with an object, that we would need $4$ times as much force to achieve the acceleration. This is certainly not true in our universe today, so we cannot say this law would be found in a similar universe.



There is also the question of the relation between inertial mass and gravitational mass, which we take to be equal. i.e. we know the force of gravity to be proportional to the mass of the object that gravity is acting on. Therefore, if forces behaved like $F=km^2a$, then we would find that, near the surface of the earth, that the acceleration due to gravity would actually depend on the mass of the object! Certainly this is not the same as our universe.



I understand the other answers' attempts to try and teach a lesson about definitions in physics, and how there are some things that are around purely by convention. But this is not one of those cases I believe. The OP is starting from $m$ means mass. So we should start there too and then see what that means for the proposed new "force laws".



This would be like me saying that it would be fine to take what we understand kinetic energy to be and rewrite it as $mv$. Sure, I could say that I have "redefined" the velocity to be $v'=sqrt2 v$. And this is valid mathematically if we stay consistent with this in the rest of our equations. But if I start off by saying "let $v$ be the velocity of the object, i.e. the rate of change of displacement", then I cannot "redefine" my equations so that kinetic energy is $mv$.






share|cite|improve this answer









$endgroup$












  • $begingroup$
    How would $g$ depend on $m$ here (for $F=km^2a$)?
    $endgroup$
    – KV18
    4 hours ago










  • $begingroup$
    @KV18 If you assumed that near the surface of the earth that the force of gravity is $mg$, then you just set up the equation: $km^2a=mg$ and solve: $a=g/km$ i.e. a more massive object would experience a smaller acceleration, which would make sense for this model of newtons second law
    $endgroup$
    – Aaron Stevens
    4 hours ago










  • $begingroup$
    But will not the force of gravity itself be $F=km^2g$?
    $endgroup$
    – KV18
    4 hours ago


















3












$begingroup$

"Force" is just a word. You don't need it at all to do Newtonian mechanics, if you start from conservation of energy and momentum and the idea of gravitational potential energy. "Force" is then just a mathematical invention to keep track of how the momentum is transferred between different bits of stuff, when things move around.



The world doesn't change just because humans invent random new words.






share|cite|improve this answer









$endgroup$












  • $begingroup$
    I do not understand the down votes to this answer. They only make clear that who gave them does not know anything about the point of view leaded by Kirchhoff, Mach, Hertz in the nineteenth century and nowadays present, anonimously, in several Mechanics textbooks (basically all of them which write that F=ma is a definition of F).
    $endgroup$
    – GiorgioP
    8 hours ago










  • $begingroup$
    @GiorgioP not only this one. It's strange that three answers had +2/-2 when I opened this page, and others 0/0.
    $endgroup$
    – Ruslan
    6 hours ago











  • $begingroup$
    @GiorgioP Down votes aren't only for answers that are incorrect or that people do not agree with
    $endgroup$
    – Aaron Stevens
    5 hours ago



















1












$begingroup$

Then we would recall mass as $m' = m^2$ or $m' = 2m $. The whole idea of Newton's second law is that force and acceleration are proportional. We can then define inertial mass as the constant factor between them. So the world would look exactly the same.






share|cite|improve this answer









$endgroup$




















    1












    $begingroup$

    Writing $F=kma$ and putting $k=1$ or $k=2,3...$ will not change the consequences of how things would actually behave.



    All it does is put up a scaling factor to the value. But due to convenience, we agree on $k=1$ implying $ F=ma$.



    Writing $a to 2a$ instead does not change anything but the convention you have to abide by, for it is the same argument as writing $kma=F$.



    If $k=m$ as you have included ($F=m^2a$), things would be the same if you assume $m^'=m^2$ and assume that $m^'$ is the actual mass.



    Besides, $m^'$ is meant to be a constant of proportionality, and so it makes no difference to the consequences of physics. $F=ma$ is based on the understanding that $F$ is proportional to $a$.






    share|cite|improve this answer











    $endgroup$








    • 2




      $begingroup$
      Writing $k = m$ in $F = kma$ would change consequences of how things behave, at least in our current system, no?
      $endgroup$
      – JMac
      9 hours ago










    • $begingroup$
      No. If $a$ becomes $2a$, then we are basically using a "different" number system. I meant that the physics does not change, only a modification to the math - like using a shifted coordinate system where $X=x+2$ - kind of like that.
      $endgroup$
      – KV18
      8 hours ago






    • 1




      $begingroup$
      $k = m$ doesn't make $a$ become $2a$ though, it makes $F = m^2 a$.
      $endgroup$
      – JMac
      8 hours ago










    • $begingroup$
      I only meant $k$ for integers.
      $endgroup$
      – KV18
      8 hours ago






    • 1




      $begingroup$
      The question uses a non-integer $k$ as an example.
      $endgroup$
      – JMac
      8 hours ago


















    1












    $begingroup$

    That is rather interesting, I've often thought of what would happen if you change a known relationship. If you doubled the mass, in your mass squared relationship, it would require four times the force to accelerate it the same. Another interesting change of relationship that happens in the real world is a rate of acceleration that increases or decreases over time. Try distance equals time cubed.






    share|cite|improve this answer











    $endgroup$












    • $begingroup$
      This does not provide an answer to the question. Once you have sufficient reputation you will be able to comment on any post; instead, provide answers that don't require clarification from the asker. - From Review
      $endgroup$
      – Cosmas Zachos
      2 hours ago


















    1












    $begingroup$

    One of the things that allowed us to invent and understand Newton's laws, which govern the motion of classical objects, is that they have the mathematical property of linearity. Linearity is a class of mathematical behaviors that people tend to be good at. If there are two or more forces acting on an object, the net force is the simple vector sum of the forces. If there is a system made of several masses, the total mass is the simple arithmetic sum of the individual masses.



    For example, an Aristotelian misconception that was properly refuted until the Renaissance is that "heavier objects fall faster." An argument against this, which I believe was due to Galileo, is a thought experiment, as follows. Imagine three identical cannonballs dropped from the same height: they should fall at the same rate because they all have the same mass. But tie two of them together with a low-mass connector, like glue or silk, and you have two objects whose masses are different by a factor of two. Can connecting two heavy masses with a silk thread change their fall time by a factor of two? It's a clever thought experiment.



    In your formulation, two identical masses $m$ accelerating in tandem with the same acceleration $a$ would have the same amount of force,



    $$
    F = m^2 a,
    $$



    acting on them. But if you tied them together so that you had a single object with mass $2m$ being acted upon by a force $2F$, then you would have



    $$
    2F = (2m)^2 a
    $$



    which predicts a different, smaller acceleration. Either you'd have to come up with an additional non-linear rule for adding forces, or you'd be describing a world that is different from the one where we live.






    share|cite|improve this answer









    $endgroup$




















      0












      $begingroup$

      No matter the transformations you make into Newton's second law, as long as it's reverse transformation admits $rm a$ to be isolated.



      $F(r,v,t)$ in Newton's second law as expressed by equation
      $$ F(r,v,t)=ma tag1$$



      Is a very abstract concept that must be adapted to solve the observed motions in the universe. More importantly, $F(r,v,t)$ is an abstraction of some kind of position, time, and velocity function that satisfy the experimental observations. Since equation (1) can be written



      $$F'(r,v,t)=a tag2$$



      and $F'(r,v,t)$ has no form a priori, any transformation in (1) that can be reversed into (2), will give the same results as Newton's second law. However, What one would call force is not necessarily what we currently call it but they should be easily converted into one another.



      For example, lets analyze the definition of gravitational force in the advent that Newton's second law were



      $$F(x,v,t)=2ma^2 tag3$$



      In this particular case, the definition of gravitational force $F_G$ here must satisfy (at least) the following observed facts:



      1. (By Galilee): “The acceleration of a falling body doesn’t depend on its mass”;

      2. (By Kepler): “The motion of planets around the sun are ellipses…”

      The simplest definition of $F_G$ that satisfies both conditions is



      $$F_G=frac4m^2r^4 tag4$$



      However, with time, people would realize that $sqrtF_G$ was actually a better physical quantity, since more intuitive generalization of it, based on geometrical insights like Gauss law would arise. The same goes for the use of $m$ instead of $2m$.






      share|cite|improve this answer











      $endgroup$




















        0












        $begingroup$

        I have not read all the answers but most of them are conceptually wrong. Newton's second law does not state that force is proportional to acceleration. Mass is not just a proportionality constant.



        Newton's second law of motion states that




        The rate of change of motion(momentum) is directly proportional to the
        motive force and the direction of the change is parallel to the force.




        Hence instead of $Falpha a$ we have $vecFalpha fracdvecpdt$. Changing the proportionality into equality
        $$vecF = kfracd(mvecv)dt$$
        Solving through
        $$vecF = kmfracdvecvdt + kvecvfracdmdt$$
        In most cases mass of system is constant and hence the second term becomes zero, but the most general form of second law is as given above.



        Now returning to the question. What if $k$ is something other than $1$. That will not change the physics. The only difference will be that accelerating objects will be harder (if $k>1$).



        The other question is invalid because $m²$ is not possible since $vecp=mvecv$. One can again ask why is momentum not $m²vecv$? You can consider momentum as $m²vecv$ just don't call $m$ as the mass, since momentum is defined that way.






        share|cite|improve this answer









        $endgroup$












        • $begingroup$
          Consider using propto in place of alpha.
          $endgroup$
          – rob
          13 mins ago











        Your Answer








        StackExchange.ready(function()
        var channelOptions =
        tags: "".split(" "),
        id: "151"
        ;
        initTagRenderer("".split(" "), "".split(" "), channelOptions);

        StackExchange.using("externalEditor", function()
        // Have to fire editor after snippets, if snippets enabled
        if (StackExchange.settings.snippets.snippetsEnabled)
        StackExchange.using("snippets", function()
        createEditor();
        );

        else
        createEditor();

        );

        function createEditor()
        StackExchange.prepareEditor(
        heartbeatType: 'answer',
        autoActivateHeartbeat: false,
        convertImagesToLinks: false,
        noModals: true,
        showLowRepImageUploadWarning: true,
        reputationToPostImages: null,
        bindNavPrevention: true,
        postfix: "",
        imageUploader:
        brandingHtml: "Powered by u003ca class="icon-imgur-white" href="https://imgur.com/"u003eu003c/au003e",
        contentPolicyHtml: "User contributions licensed under u003ca href="https://creativecommons.org/licenses/by-sa/3.0/"u003ecc by-sa 3.0 with attribution requiredu003c/au003e u003ca href="https://stackoverflow.com/legal/content-policy"u003e(content policy)u003c/au003e",
        allowUrls: true
        ,
        noCode: true, onDemand: true,
        discardSelector: ".discard-answer"
        ,immediatelyShowMarkdownHelp:true
        );



        );






        pooja paliwal is a new contributor. Be nice, and check out our Code of Conduct.









        draft saved

        draft discarded


















        StackExchange.ready(
        function ()
        StackExchange.openid.initPostLogin('.new-post-login', 'https%3a%2f%2fphysics.stackexchange.com%2fquestions%2f475733%2fwhat-if-force-was-not-mass-times-acceleration%23new-answer', 'question_page');

        );

        Post as a guest















        Required, but never shown

























        9 Answers
        9






        active

        oldest

        votes








        9 Answers
        9






        active

        oldest

        votes









        active

        oldest

        votes






        active

        oldest

        votes









        2












        $begingroup$

        It wouldn't change anything. We would just have defined mass differently. What is relevant about Newton's 2nd law is not $m$ but rather the fact that $F$ and $a$ are proportional, $Fpropto a$.



        The proportionality constant can then be called $m$ or $2m$ or $m^2$. That doesn't matter for the relationship that this law describes.



        Newton's 2nd law defines mass (inertial mass) as the proportionality constant in $F=ma$. Had we decided upon $F=2ma$, then we would just have defined mass as half of the proportionality constant. It would change our language regarding mass (you would only weigh 40 kg rather than 80 kg i.e.), but it wouldn't change the way the world works.



        If you on the other hand had asked about $F=ma^2$ or so, then you are messing with the world entirely with disastrous consequences that we can only try to guess as a result!






        share|cite|improve this answer









        $endgroup$








        • 6




          $begingroup$
          It would change something though as we wouldn’t have eg universal acceleration. Galileo would not have found the balls hit the ground at the same time when dropped from the tower ;)
          $endgroup$
          – innisfree
          8 hours ago






        • 8




          $begingroup$
          I disagree that it would be so simple. It would also have the consequence that when putting two objects together, we would have to add their masses squared instead of their masses, which seems pretty strange. Clearly what we now call mass is a better option than the others.
          $endgroup$
          – Javier
          8 hours ago






        • 1




          $begingroup$
          We’d also have to use some different units or fix the units somehow, eg introduce another dimensionful constant of nature or maybe just change the units of Newton’s constant $G$
          $endgroup$
          – innisfree
          8 hours ago






        • 3




          $begingroup$
          It would be nice if this answer (which is the accepted one) made mention of gravitational mass (and other non-inertial properties we expect mass to have, like additivity) and how they change.
          $endgroup$
          – jacob1729
          8 hours ago






        • 1




          $begingroup$
          If it was $m^2$ then it would definitely change something if the $m$ also refers to the mass associated with the force of gravity
          $endgroup$
          – Aaron Stevens
          5 hours ago
















        2












        $begingroup$

        It wouldn't change anything. We would just have defined mass differently. What is relevant about Newton's 2nd law is not $m$ but rather the fact that $F$ and $a$ are proportional, $Fpropto a$.



        The proportionality constant can then be called $m$ or $2m$ or $m^2$. That doesn't matter for the relationship that this law describes.



        Newton's 2nd law defines mass (inertial mass) as the proportionality constant in $F=ma$. Had we decided upon $F=2ma$, then we would just have defined mass as half of the proportionality constant. It would change our language regarding mass (you would only weigh 40 kg rather than 80 kg i.e.), but it wouldn't change the way the world works.



        If you on the other hand had asked about $F=ma^2$ or so, then you are messing with the world entirely with disastrous consequences that we can only try to guess as a result!






        share|cite|improve this answer









        $endgroup$








        • 6




          $begingroup$
          It would change something though as we wouldn’t have eg universal acceleration. Galileo would not have found the balls hit the ground at the same time when dropped from the tower ;)
          $endgroup$
          – innisfree
          8 hours ago






        • 8




          $begingroup$
          I disagree that it would be so simple. It would also have the consequence that when putting two objects together, we would have to add their masses squared instead of their masses, which seems pretty strange. Clearly what we now call mass is a better option than the others.
          $endgroup$
          – Javier
          8 hours ago






        • 1




          $begingroup$
          We’d also have to use some different units or fix the units somehow, eg introduce another dimensionful constant of nature or maybe just change the units of Newton’s constant $G$
          $endgroup$
          – innisfree
          8 hours ago






        • 3




          $begingroup$
          It would be nice if this answer (which is the accepted one) made mention of gravitational mass (and other non-inertial properties we expect mass to have, like additivity) and how they change.
          $endgroup$
          – jacob1729
          8 hours ago






        • 1




          $begingroup$
          If it was $m^2$ then it would definitely change something if the $m$ also refers to the mass associated with the force of gravity
          $endgroup$
          – Aaron Stevens
          5 hours ago














        2












        2








        2





        $begingroup$

        It wouldn't change anything. We would just have defined mass differently. What is relevant about Newton's 2nd law is not $m$ but rather the fact that $F$ and $a$ are proportional, $Fpropto a$.



        The proportionality constant can then be called $m$ or $2m$ or $m^2$. That doesn't matter for the relationship that this law describes.



        Newton's 2nd law defines mass (inertial mass) as the proportionality constant in $F=ma$. Had we decided upon $F=2ma$, then we would just have defined mass as half of the proportionality constant. It would change our language regarding mass (you would only weigh 40 kg rather than 80 kg i.e.), but it wouldn't change the way the world works.



        If you on the other hand had asked about $F=ma^2$ or so, then you are messing with the world entirely with disastrous consequences that we can only try to guess as a result!






        share|cite|improve this answer









        $endgroup$



        It wouldn't change anything. We would just have defined mass differently. What is relevant about Newton's 2nd law is not $m$ but rather the fact that $F$ and $a$ are proportional, $Fpropto a$.



        The proportionality constant can then be called $m$ or $2m$ or $m^2$. That doesn't matter for the relationship that this law describes.



        Newton's 2nd law defines mass (inertial mass) as the proportionality constant in $F=ma$. Had we decided upon $F=2ma$, then we would just have defined mass as half of the proportionality constant. It would change our language regarding mass (you would only weigh 40 kg rather than 80 kg i.e.), but it wouldn't change the way the world works.



        If you on the other hand had asked about $F=ma^2$ or so, then you are messing with the world entirely with disastrous consequences that we can only try to guess as a result!







        share|cite|improve this answer












        share|cite|improve this answer



        share|cite|improve this answer










        answered 9 hours ago









        SteevenSteeven

        28.1k866114




        28.1k866114







        • 6




          $begingroup$
          It would change something though as we wouldn’t have eg universal acceleration. Galileo would not have found the balls hit the ground at the same time when dropped from the tower ;)
          $endgroup$
          – innisfree
          8 hours ago






        • 8




          $begingroup$
          I disagree that it would be so simple. It would also have the consequence that when putting two objects together, we would have to add their masses squared instead of their masses, which seems pretty strange. Clearly what we now call mass is a better option than the others.
          $endgroup$
          – Javier
          8 hours ago






        • 1




          $begingroup$
          We’d also have to use some different units or fix the units somehow, eg introduce another dimensionful constant of nature or maybe just change the units of Newton’s constant $G$
          $endgroup$
          – innisfree
          8 hours ago






        • 3




          $begingroup$
          It would be nice if this answer (which is the accepted one) made mention of gravitational mass (and other non-inertial properties we expect mass to have, like additivity) and how they change.
          $endgroup$
          – jacob1729
          8 hours ago






        • 1




          $begingroup$
          If it was $m^2$ then it would definitely change something if the $m$ also refers to the mass associated with the force of gravity
          $endgroup$
          – Aaron Stevens
          5 hours ago













        • 6




          $begingroup$
          It would change something though as we wouldn’t have eg universal acceleration. Galileo would not have found the balls hit the ground at the same time when dropped from the tower ;)
          $endgroup$
          – innisfree
          8 hours ago






        • 8




          $begingroup$
          I disagree that it would be so simple. It would also have the consequence that when putting two objects together, we would have to add their masses squared instead of their masses, which seems pretty strange. Clearly what we now call mass is a better option than the others.
          $endgroup$
          – Javier
          8 hours ago






        • 1




          $begingroup$
          We’d also have to use some different units or fix the units somehow, eg introduce another dimensionful constant of nature or maybe just change the units of Newton’s constant $G$
          $endgroup$
          – innisfree
          8 hours ago






        • 3




          $begingroup$
          It would be nice if this answer (which is the accepted one) made mention of gravitational mass (and other non-inertial properties we expect mass to have, like additivity) and how they change.
          $endgroup$
          – jacob1729
          8 hours ago






        • 1




          $begingroup$
          If it was $m^2$ then it would definitely change something if the $m$ also refers to the mass associated with the force of gravity
          $endgroup$
          – Aaron Stevens
          5 hours ago








        6




        6




        $begingroup$
        It would change something though as we wouldn’t have eg universal acceleration. Galileo would not have found the balls hit the ground at the same time when dropped from the tower ;)
        $endgroup$
        – innisfree
        8 hours ago




        $begingroup$
        It would change something though as we wouldn’t have eg universal acceleration. Galileo would not have found the balls hit the ground at the same time when dropped from the tower ;)
        $endgroup$
        – innisfree
        8 hours ago




        8




        8




        $begingroup$
        I disagree that it would be so simple. It would also have the consequence that when putting two objects together, we would have to add their masses squared instead of their masses, which seems pretty strange. Clearly what we now call mass is a better option than the others.
        $endgroup$
        – Javier
        8 hours ago




        $begingroup$
        I disagree that it would be so simple. It would also have the consequence that when putting two objects together, we would have to add their masses squared instead of their masses, which seems pretty strange. Clearly what we now call mass is a better option than the others.
        $endgroup$
        – Javier
        8 hours ago




        1




        1




        $begingroup$
        We’d also have to use some different units or fix the units somehow, eg introduce another dimensionful constant of nature or maybe just change the units of Newton’s constant $G$
        $endgroup$
        – innisfree
        8 hours ago




        $begingroup$
        We’d also have to use some different units or fix the units somehow, eg introduce another dimensionful constant of nature or maybe just change the units of Newton’s constant $G$
        $endgroup$
        – innisfree
        8 hours ago




        3




        3




        $begingroup$
        It would be nice if this answer (which is the accepted one) made mention of gravitational mass (and other non-inertial properties we expect mass to have, like additivity) and how they change.
        $endgroup$
        – jacob1729
        8 hours ago




        $begingroup$
        It would be nice if this answer (which is the accepted one) made mention of gravitational mass (and other non-inertial properties we expect mass to have, like additivity) and how they change.
        $endgroup$
        – jacob1729
        8 hours ago




        1




        1




        $begingroup$
        If it was $m^2$ then it would definitely change something if the $m$ also refers to the mass associated with the force of gravity
        $endgroup$
        – Aaron Stevens
        5 hours ago





        $begingroup$
        If it was $m^2$ then it would definitely change something if the $m$ also refers to the mass associated with the force of gravity
        $endgroup$
        – Aaron Stevens
        5 hours ago












        6












        $begingroup$

        In contrast to the other answers, I am going to go a different direction. This is not a question of mathematics, where we can just say "oh yeah just redefine your variables." This is a physics question.



        Going further, The OP is assuming $m$ stands for the mass of an object, not some arbitrary proportionality constant that ends up being the mass. Therefore, having a law like $F=km^2a$ ($k$ as just some constant for unit's sake) would not leave the universe unchanged. This is saying, for example, that if we double the "amount of stuff" associated with an object, that we would need $4$ times as much force to achieve the acceleration. This is certainly not true in our universe today, so we cannot say this law would be found in a similar universe.



        There is also the question of the relation between inertial mass and gravitational mass, which we take to be equal. i.e. we know the force of gravity to be proportional to the mass of the object that gravity is acting on. Therefore, if forces behaved like $F=km^2a$, then we would find that, near the surface of the earth, that the acceleration due to gravity would actually depend on the mass of the object! Certainly this is not the same as our universe.



        I understand the other answers' attempts to try and teach a lesson about definitions in physics, and how there are some things that are around purely by convention. But this is not one of those cases I believe. The OP is starting from $m$ means mass. So we should start there too and then see what that means for the proposed new "force laws".



        This would be like me saying that it would be fine to take what we understand kinetic energy to be and rewrite it as $mv$. Sure, I could say that I have "redefined" the velocity to be $v'=sqrt2 v$. And this is valid mathematically if we stay consistent with this in the rest of our equations. But if I start off by saying "let $v$ be the velocity of the object, i.e. the rate of change of displacement", then I cannot "redefine" my equations so that kinetic energy is $mv$.






        share|cite|improve this answer









        $endgroup$












        • $begingroup$
          How would $g$ depend on $m$ here (for $F=km^2a$)?
          $endgroup$
          – KV18
          4 hours ago










        • $begingroup$
          @KV18 If you assumed that near the surface of the earth that the force of gravity is $mg$, then you just set up the equation: $km^2a=mg$ and solve: $a=g/km$ i.e. a more massive object would experience a smaller acceleration, which would make sense for this model of newtons second law
          $endgroup$
          – Aaron Stevens
          4 hours ago










        • $begingroup$
          But will not the force of gravity itself be $F=km^2g$?
          $endgroup$
          – KV18
          4 hours ago















        6












        $begingroup$

        In contrast to the other answers, I am going to go a different direction. This is not a question of mathematics, where we can just say "oh yeah just redefine your variables." This is a physics question.



        Going further, The OP is assuming $m$ stands for the mass of an object, not some arbitrary proportionality constant that ends up being the mass. Therefore, having a law like $F=km^2a$ ($k$ as just some constant for unit's sake) would not leave the universe unchanged. This is saying, for example, that if we double the "amount of stuff" associated with an object, that we would need $4$ times as much force to achieve the acceleration. This is certainly not true in our universe today, so we cannot say this law would be found in a similar universe.



        There is also the question of the relation between inertial mass and gravitational mass, which we take to be equal. i.e. we know the force of gravity to be proportional to the mass of the object that gravity is acting on. Therefore, if forces behaved like $F=km^2a$, then we would find that, near the surface of the earth, that the acceleration due to gravity would actually depend on the mass of the object! Certainly this is not the same as our universe.



        I understand the other answers' attempts to try and teach a lesson about definitions in physics, and how there are some things that are around purely by convention. But this is not one of those cases I believe. The OP is starting from $m$ means mass. So we should start there too and then see what that means for the proposed new "force laws".



        This would be like me saying that it would be fine to take what we understand kinetic energy to be and rewrite it as $mv$. Sure, I could say that I have "redefined" the velocity to be $v'=sqrt2 v$. And this is valid mathematically if we stay consistent with this in the rest of our equations. But if I start off by saying "let $v$ be the velocity of the object, i.e. the rate of change of displacement", then I cannot "redefine" my equations so that kinetic energy is $mv$.






        share|cite|improve this answer









        $endgroup$












        • $begingroup$
          How would $g$ depend on $m$ here (for $F=km^2a$)?
          $endgroup$
          – KV18
          4 hours ago










        • $begingroup$
          @KV18 If you assumed that near the surface of the earth that the force of gravity is $mg$, then you just set up the equation: $km^2a=mg$ and solve: $a=g/km$ i.e. a more massive object would experience a smaller acceleration, which would make sense for this model of newtons second law
          $endgroup$
          – Aaron Stevens
          4 hours ago










        • $begingroup$
          But will not the force of gravity itself be $F=km^2g$?
          $endgroup$
          – KV18
          4 hours ago













        6












        6








        6





        $begingroup$

        In contrast to the other answers, I am going to go a different direction. This is not a question of mathematics, where we can just say "oh yeah just redefine your variables." This is a physics question.



        Going further, The OP is assuming $m$ stands for the mass of an object, not some arbitrary proportionality constant that ends up being the mass. Therefore, having a law like $F=km^2a$ ($k$ as just some constant for unit's sake) would not leave the universe unchanged. This is saying, for example, that if we double the "amount of stuff" associated with an object, that we would need $4$ times as much force to achieve the acceleration. This is certainly not true in our universe today, so we cannot say this law would be found in a similar universe.



        There is also the question of the relation between inertial mass and gravitational mass, which we take to be equal. i.e. we know the force of gravity to be proportional to the mass of the object that gravity is acting on. Therefore, if forces behaved like $F=km^2a$, then we would find that, near the surface of the earth, that the acceleration due to gravity would actually depend on the mass of the object! Certainly this is not the same as our universe.



        I understand the other answers' attempts to try and teach a lesson about definitions in physics, and how there are some things that are around purely by convention. But this is not one of those cases I believe. The OP is starting from $m$ means mass. So we should start there too and then see what that means for the proposed new "force laws".



        This would be like me saying that it would be fine to take what we understand kinetic energy to be and rewrite it as $mv$. Sure, I could say that I have "redefined" the velocity to be $v'=sqrt2 v$. And this is valid mathematically if we stay consistent with this in the rest of our equations. But if I start off by saying "let $v$ be the velocity of the object, i.e. the rate of change of displacement", then I cannot "redefine" my equations so that kinetic energy is $mv$.






        share|cite|improve this answer









        $endgroup$



        In contrast to the other answers, I am going to go a different direction. This is not a question of mathematics, where we can just say "oh yeah just redefine your variables." This is a physics question.



        Going further, The OP is assuming $m$ stands for the mass of an object, not some arbitrary proportionality constant that ends up being the mass. Therefore, having a law like $F=km^2a$ ($k$ as just some constant for unit's sake) would not leave the universe unchanged. This is saying, for example, that if we double the "amount of stuff" associated with an object, that we would need $4$ times as much force to achieve the acceleration. This is certainly not true in our universe today, so we cannot say this law would be found in a similar universe.



        There is also the question of the relation between inertial mass and gravitational mass, which we take to be equal. i.e. we know the force of gravity to be proportional to the mass of the object that gravity is acting on. Therefore, if forces behaved like $F=km^2a$, then we would find that, near the surface of the earth, that the acceleration due to gravity would actually depend on the mass of the object! Certainly this is not the same as our universe.



        I understand the other answers' attempts to try and teach a lesson about definitions in physics, and how there are some things that are around purely by convention. But this is not one of those cases I believe. The OP is starting from $m$ means mass. So we should start there too and then see what that means for the proposed new "force laws".



        This would be like me saying that it would be fine to take what we understand kinetic energy to be and rewrite it as $mv$. Sure, I could say that I have "redefined" the velocity to be $v'=sqrt2 v$. And this is valid mathematically if we stay consistent with this in the rest of our equations. But if I start off by saying "let $v$ be the velocity of the object, i.e. the rate of change of displacement", then I cannot "redefine" my equations so that kinetic energy is $mv$.







        share|cite|improve this answer












        share|cite|improve this answer



        share|cite|improve this answer










        answered 5 hours ago









        Aaron StevensAaron Stevens

        15.7k42557




        15.7k42557











        • $begingroup$
          How would $g$ depend on $m$ here (for $F=km^2a$)?
          $endgroup$
          – KV18
          4 hours ago










        • $begingroup$
          @KV18 If you assumed that near the surface of the earth that the force of gravity is $mg$, then you just set up the equation: $km^2a=mg$ and solve: $a=g/km$ i.e. a more massive object would experience a smaller acceleration, which would make sense for this model of newtons second law
          $endgroup$
          – Aaron Stevens
          4 hours ago










        • $begingroup$
          But will not the force of gravity itself be $F=km^2g$?
          $endgroup$
          – KV18
          4 hours ago
















        • $begingroup$
          How would $g$ depend on $m$ here (for $F=km^2a$)?
          $endgroup$
          – KV18
          4 hours ago










        • $begingroup$
          @KV18 If you assumed that near the surface of the earth that the force of gravity is $mg$, then you just set up the equation: $km^2a=mg$ and solve: $a=g/km$ i.e. a more massive object would experience a smaller acceleration, which would make sense for this model of newtons second law
          $endgroup$
          – Aaron Stevens
          4 hours ago










        • $begingroup$
          But will not the force of gravity itself be $F=km^2g$?
          $endgroup$
          – KV18
          4 hours ago















        $begingroup$
        How would $g$ depend on $m$ here (for $F=km^2a$)?
        $endgroup$
        – KV18
        4 hours ago




        $begingroup$
        How would $g$ depend on $m$ here (for $F=km^2a$)?
        $endgroup$
        – KV18
        4 hours ago












        $begingroup$
        @KV18 If you assumed that near the surface of the earth that the force of gravity is $mg$, then you just set up the equation: $km^2a=mg$ and solve: $a=g/km$ i.e. a more massive object would experience a smaller acceleration, which would make sense for this model of newtons second law
        $endgroup$
        – Aaron Stevens
        4 hours ago




        $begingroup$
        @KV18 If you assumed that near the surface of the earth that the force of gravity is $mg$, then you just set up the equation: $km^2a=mg$ and solve: $a=g/km$ i.e. a more massive object would experience a smaller acceleration, which would make sense for this model of newtons second law
        $endgroup$
        – Aaron Stevens
        4 hours ago












        $begingroup$
        But will not the force of gravity itself be $F=km^2g$?
        $endgroup$
        – KV18
        4 hours ago




        $begingroup$
        But will not the force of gravity itself be $F=km^2g$?
        $endgroup$
        – KV18
        4 hours ago











        3












        $begingroup$

        "Force" is just a word. You don't need it at all to do Newtonian mechanics, if you start from conservation of energy and momentum and the idea of gravitational potential energy. "Force" is then just a mathematical invention to keep track of how the momentum is transferred between different bits of stuff, when things move around.



        The world doesn't change just because humans invent random new words.






        share|cite|improve this answer









        $endgroup$












        • $begingroup$
          I do not understand the down votes to this answer. They only make clear that who gave them does not know anything about the point of view leaded by Kirchhoff, Mach, Hertz in the nineteenth century and nowadays present, anonimously, in several Mechanics textbooks (basically all of them which write that F=ma is a definition of F).
          $endgroup$
          – GiorgioP
          8 hours ago










        • $begingroup$
          @GiorgioP not only this one. It's strange that three answers had +2/-2 when I opened this page, and others 0/0.
          $endgroup$
          – Ruslan
          6 hours ago











        • $begingroup$
          @GiorgioP Down votes aren't only for answers that are incorrect or that people do not agree with
          $endgroup$
          – Aaron Stevens
          5 hours ago
















        3












        $begingroup$

        "Force" is just a word. You don't need it at all to do Newtonian mechanics, if you start from conservation of energy and momentum and the idea of gravitational potential energy. "Force" is then just a mathematical invention to keep track of how the momentum is transferred between different bits of stuff, when things move around.



        The world doesn't change just because humans invent random new words.






        share|cite|improve this answer









        $endgroup$












        • $begingroup$
          I do not understand the down votes to this answer. They only make clear that who gave them does not know anything about the point of view leaded by Kirchhoff, Mach, Hertz in the nineteenth century and nowadays present, anonimously, in several Mechanics textbooks (basically all of them which write that F=ma is a definition of F).
          $endgroup$
          – GiorgioP
          8 hours ago










        • $begingroup$
          @GiorgioP not only this one. It's strange that three answers had +2/-2 when I opened this page, and others 0/0.
          $endgroup$
          – Ruslan
          6 hours ago











        • $begingroup$
          @GiorgioP Down votes aren't only for answers that are incorrect or that people do not agree with
          $endgroup$
          – Aaron Stevens
          5 hours ago














        3












        3








        3





        $begingroup$

        "Force" is just a word. You don't need it at all to do Newtonian mechanics, if you start from conservation of energy and momentum and the idea of gravitational potential energy. "Force" is then just a mathematical invention to keep track of how the momentum is transferred between different bits of stuff, when things move around.



        The world doesn't change just because humans invent random new words.






        share|cite|improve this answer









        $endgroup$



        "Force" is just a word. You don't need it at all to do Newtonian mechanics, if you start from conservation of energy and momentum and the idea of gravitational potential energy. "Force" is then just a mathematical invention to keep track of how the momentum is transferred between different bits of stuff, when things move around.



        The world doesn't change just because humans invent random new words.







        share|cite|improve this answer












        share|cite|improve this answer



        share|cite|improve this answer










        answered 8 hours ago









        alephzeroalephzero

        5,92321121




        5,92321121











        • $begingroup$
          I do not understand the down votes to this answer. They only make clear that who gave them does not know anything about the point of view leaded by Kirchhoff, Mach, Hertz in the nineteenth century and nowadays present, anonimously, in several Mechanics textbooks (basically all of them which write that F=ma is a definition of F).
          $endgroup$
          – GiorgioP
          8 hours ago










        • $begingroup$
          @GiorgioP not only this one. It's strange that three answers had +2/-2 when I opened this page, and others 0/0.
          $endgroup$
          – Ruslan
          6 hours ago











        • $begingroup$
          @GiorgioP Down votes aren't only for answers that are incorrect or that people do not agree with
          $endgroup$
          – Aaron Stevens
          5 hours ago

















        • $begingroup$
          I do not understand the down votes to this answer. They only make clear that who gave them does not know anything about the point of view leaded by Kirchhoff, Mach, Hertz in the nineteenth century and nowadays present, anonimously, in several Mechanics textbooks (basically all of them which write that F=ma is a definition of F).
          $endgroup$
          – GiorgioP
          8 hours ago










        • $begingroup$
          @GiorgioP not only this one. It's strange that three answers had +2/-2 when I opened this page, and others 0/0.
          $endgroup$
          – Ruslan
          6 hours ago











        • $begingroup$
          @GiorgioP Down votes aren't only for answers that are incorrect or that people do not agree with
          $endgroup$
          – Aaron Stevens
          5 hours ago
















        $begingroup$
        I do not understand the down votes to this answer. They only make clear that who gave them does not know anything about the point of view leaded by Kirchhoff, Mach, Hertz in the nineteenth century and nowadays present, anonimously, in several Mechanics textbooks (basically all of them which write that F=ma is a definition of F).
        $endgroup$
        – GiorgioP
        8 hours ago




        $begingroup$
        I do not understand the down votes to this answer. They only make clear that who gave them does not know anything about the point of view leaded by Kirchhoff, Mach, Hertz in the nineteenth century and nowadays present, anonimously, in several Mechanics textbooks (basically all of them which write that F=ma is a definition of F).
        $endgroup$
        – GiorgioP
        8 hours ago












        $begingroup$
        @GiorgioP not only this one. It's strange that three answers had +2/-2 when I opened this page, and others 0/0.
        $endgroup$
        – Ruslan
        6 hours ago





        $begingroup$
        @GiorgioP not only this one. It's strange that three answers had +2/-2 when I opened this page, and others 0/0.
        $endgroup$
        – Ruslan
        6 hours ago













        $begingroup$
        @GiorgioP Down votes aren't only for answers that are incorrect or that people do not agree with
        $endgroup$
        – Aaron Stevens
        5 hours ago





        $begingroup$
        @GiorgioP Down votes aren't only for answers that are incorrect or that people do not agree with
        $endgroup$
        – Aaron Stevens
        5 hours ago












        1












        $begingroup$

        Then we would recall mass as $m' = m^2$ or $m' = 2m $. The whole idea of Newton's second law is that force and acceleration are proportional. We can then define inertial mass as the constant factor between them. So the world would look exactly the same.






        share|cite|improve this answer









        $endgroup$

















          1












          $begingroup$

          Then we would recall mass as $m' = m^2$ or $m' = 2m $. The whole idea of Newton's second law is that force and acceleration are proportional. We can then define inertial mass as the constant factor between them. So the world would look exactly the same.






          share|cite|improve this answer









          $endgroup$















            1












            1








            1





            $begingroup$

            Then we would recall mass as $m' = m^2$ or $m' = 2m $. The whole idea of Newton's second law is that force and acceleration are proportional. We can then define inertial mass as the constant factor between them. So the world would look exactly the same.






            share|cite|improve this answer









            $endgroup$



            Then we would recall mass as $m' = m^2$ or $m' = 2m $. The whole idea of Newton's second law is that force and acceleration are proportional. We can then define inertial mass as the constant factor between them. So the world would look exactly the same.







            share|cite|improve this answer












            share|cite|improve this answer



            share|cite|improve this answer










            answered 9 hours ago









            Ollie113Ollie113

            777




            777





















                1












                $begingroup$

                Writing $F=kma$ and putting $k=1$ or $k=2,3...$ will not change the consequences of how things would actually behave.



                All it does is put up a scaling factor to the value. But due to convenience, we agree on $k=1$ implying $ F=ma$.



                Writing $a to 2a$ instead does not change anything but the convention you have to abide by, for it is the same argument as writing $kma=F$.



                If $k=m$ as you have included ($F=m^2a$), things would be the same if you assume $m^'=m^2$ and assume that $m^'$ is the actual mass.



                Besides, $m^'$ is meant to be a constant of proportionality, and so it makes no difference to the consequences of physics. $F=ma$ is based on the understanding that $F$ is proportional to $a$.






                share|cite|improve this answer











                $endgroup$








                • 2




                  $begingroup$
                  Writing $k = m$ in $F = kma$ would change consequences of how things behave, at least in our current system, no?
                  $endgroup$
                  – JMac
                  9 hours ago










                • $begingroup$
                  No. If $a$ becomes $2a$, then we are basically using a "different" number system. I meant that the physics does not change, only a modification to the math - like using a shifted coordinate system where $X=x+2$ - kind of like that.
                  $endgroup$
                  – KV18
                  8 hours ago






                • 1




                  $begingroup$
                  $k = m$ doesn't make $a$ become $2a$ though, it makes $F = m^2 a$.
                  $endgroup$
                  – JMac
                  8 hours ago










                • $begingroup$
                  I only meant $k$ for integers.
                  $endgroup$
                  – KV18
                  8 hours ago






                • 1




                  $begingroup$
                  The question uses a non-integer $k$ as an example.
                  $endgroup$
                  – JMac
                  8 hours ago















                1












                $begingroup$

                Writing $F=kma$ and putting $k=1$ or $k=2,3...$ will not change the consequences of how things would actually behave.



                All it does is put up a scaling factor to the value. But due to convenience, we agree on $k=1$ implying $ F=ma$.



                Writing $a to 2a$ instead does not change anything but the convention you have to abide by, for it is the same argument as writing $kma=F$.



                If $k=m$ as you have included ($F=m^2a$), things would be the same if you assume $m^'=m^2$ and assume that $m^'$ is the actual mass.



                Besides, $m^'$ is meant to be a constant of proportionality, and so it makes no difference to the consequences of physics. $F=ma$ is based on the understanding that $F$ is proportional to $a$.






                share|cite|improve this answer











                $endgroup$








                • 2




                  $begingroup$
                  Writing $k = m$ in $F = kma$ would change consequences of how things behave, at least in our current system, no?
                  $endgroup$
                  – JMac
                  9 hours ago










                • $begingroup$
                  No. If $a$ becomes $2a$, then we are basically using a "different" number system. I meant that the physics does not change, only a modification to the math - like using a shifted coordinate system where $X=x+2$ - kind of like that.
                  $endgroup$
                  – KV18
                  8 hours ago






                • 1




                  $begingroup$
                  $k = m$ doesn't make $a$ become $2a$ though, it makes $F = m^2 a$.
                  $endgroup$
                  – JMac
                  8 hours ago










                • $begingroup$
                  I only meant $k$ for integers.
                  $endgroup$
                  – KV18
                  8 hours ago






                • 1




                  $begingroup$
                  The question uses a non-integer $k$ as an example.
                  $endgroup$
                  – JMac
                  8 hours ago













                1












                1








                1





                $begingroup$

                Writing $F=kma$ and putting $k=1$ or $k=2,3...$ will not change the consequences of how things would actually behave.



                All it does is put up a scaling factor to the value. But due to convenience, we agree on $k=1$ implying $ F=ma$.



                Writing $a to 2a$ instead does not change anything but the convention you have to abide by, for it is the same argument as writing $kma=F$.



                If $k=m$ as you have included ($F=m^2a$), things would be the same if you assume $m^'=m^2$ and assume that $m^'$ is the actual mass.



                Besides, $m^'$ is meant to be a constant of proportionality, and so it makes no difference to the consequences of physics. $F=ma$ is based on the understanding that $F$ is proportional to $a$.






                share|cite|improve this answer











                $endgroup$



                Writing $F=kma$ and putting $k=1$ or $k=2,3...$ will not change the consequences of how things would actually behave.



                All it does is put up a scaling factor to the value. But due to convenience, we agree on $k=1$ implying $ F=ma$.



                Writing $a to 2a$ instead does not change anything but the convention you have to abide by, for it is the same argument as writing $kma=F$.



                If $k=m$ as you have included ($F=m^2a$), things would be the same if you assume $m^'=m^2$ and assume that $m^'$ is the actual mass.



                Besides, $m^'$ is meant to be a constant of proportionality, and so it makes no difference to the consequences of physics. $F=ma$ is based on the understanding that $F$ is proportional to $a$.







                share|cite|improve this answer














                share|cite|improve this answer



                share|cite|improve this answer








                edited 8 hours ago

























                answered 9 hours ago









                KV18KV18

                1,294617




                1,294617







                • 2




                  $begingroup$
                  Writing $k = m$ in $F = kma$ would change consequences of how things behave, at least in our current system, no?
                  $endgroup$
                  – JMac
                  9 hours ago










                • $begingroup$
                  No. If $a$ becomes $2a$, then we are basically using a "different" number system. I meant that the physics does not change, only a modification to the math - like using a shifted coordinate system where $X=x+2$ - kind of like that.
                  $endgroup$
                  – KV18
                  8 hours ago






                • 1




                  $begingroup$
                  $k = m$ doesn't make $a$ become $2a$ though, it makes $F = m^2 a$.
                  $endgroup$
                  – JMac
                  8 hours ago










                • $begingroup$
                  I only meant $k$ for integers.
                  $endgroup$
                  – KV18
                  8 hours ago






                • 1




                  $begingroup$
                  The question uses a non-integer $k$ as an example.
                  $endgroup$
                  – JMac
                  8 hours ago












                • 2




                  $begingroup$
                  Writing $k = m$ in $F = kma$ would change consequences of how things behave, at least in our current system, no?
                  $endgroup$
                  – JMac
                  9 hours ago










                • $begingroup$
                  No. If $a$ becomes $2a$, then we are basically using a "different" number system. I meant that the physics does not change, only a modification to the math - like using a shifted coordinate system where $X=x+2$ - kind of like that.
                  $endgroup$
                  – KV18
                  8 hours ago






                • 1




                  $begingroup$
                  $k = m$ doesn't make $a$ become $2a$ though, it makes $F = m^2 a$.
                  $endgroup$
                  – JMac
                  8 hours ago










                • $begingroup$
                  I only meant $k$ for integers.
                  $endgroup$
                  – KV18
                  8 hours ago






                • 1




                  $begingroup$
                  The question uses a non-integer $k$ as an example.
                  $endgroup$
                  – JMac
                  8 hours ago







                2




                2




                $begingroup$
                Writing $k = m$ in $F = kma$ would change consequences of how things behave, at least in our current system, no?
                $endgroup$
                – JMac
                9 hours ago




                $begingroup$
                Writing $k = m$ in $F = kma$ would change consequences of how things behave, at least in our current system, no?
                $endgroup$
                – JMac
                9 hours ago












                $begingroup$
                No. If $a$ becomes $2a$, then we are basically using a "different" number system. I meant that the physics does not change, only a modification to the math - like using a shifted coordinate system where $X=x+2$ - kind of like that.
                $endgroup$
                – KV18
                8 hours ago




                $begingroup$
                No. If $a$ becomes $2a$, then we are basically using a "different" number system. I meant that the physics does not change, only a modification to the math - like using a shifted coordinate system where $X=x+2$ - kind of like that.
                $endgroup$
                – KV18
                8 hours ago




                1




                1




                $begingroup$
                $k = m$ doesn't make $a$ become $2a$ though, it makes $F = m^2 a$.
                $endgroup$
                – JMac
                8 hours ago




                $begingroup$
                $k = m$ doesn't make $a$ become $2a$ though, it makes $F = m^2 a$.
                $endgroup$
                – JMac
                8 hours ago












                $begingroup$
                I only meant $k$ for integers.
                $endgroup$
                – KV18
                8 hours ago




                $begingroup$
                I only meant $k$ for integers.
                $endgroup$
                – KV18
                8 hours ago




                1




                1




                $begingroup$
                The question uses a non-integer $k$ as an example.
                $endgroup$
                – JMac
                8 hours ago




                $begingroup$
                The question uses a non-integer $k$ as an example.
                $endgroup$
                – JMac
                8 hours ago











                1












                $begingroup$

                That is rather interesting, I've often thought of what would happen if you change a known relationship. If you doubled the mass, in your mass squared relationship, it would require four times the force to accelerate it the same. Another interesting change of relationship that happens in the real world is a rate of acceleration that increases or decreases over time. Try distance equals time cubed.






                share|cite|improve this answer











                $endgroup$












                • $begingroup$
                  This does not provide an answer to the question. Once you have sufficient reputation you will be able to comment on any post; instead, provide answers that don't require clarification from the asker. - From Review
                  $endgroup$
                  – Cosmas Zachos
                  2 hours ago















                1












                $begingroup$

                That is rather interesting, I've often thought of what would happen if you change a known relationship. If you doubled the mass, in your mass squared relationship, it would require four times the force to accelerate it the same. Another interesting change of relationship that happens in the real world is a rate of acceleration that increases or decreases over time. Try distance equals time cubed.






                share|cite|improve this answer











                $endgroup$












                • $begingroup$
                  This does not provide an answer to the question. Once you have sufficient reputation you will be able to comment on any post; instead, provide answers that don't require clarification from the asker. - From Review
                  $endgroup$
                  – Cosmas Zachos
                  2 hours ago













                1












                1








                1





                $begingroup$

                That is rather interesting, I've often thought of what would happen if you change a known relationship. If you doubled the mass, in your mass squared relationship, it would require four times the force to accelerate it the same. Another interesting change of relationship that happens in the real world is a rate of acceleration that increases or decreases over time. Try distance equals time cubed.






                share|cite|improve this answer











                $endgroup$



                That is rather interesting, I've often thought of what would happen if you change a known relationship. If you doubled the mass, in your mass squared relationship, it would require four times the force to accelerate it the same. Another interesting change of relationship that happens in the real world is a rate of acceleration that increases or decreases over time. Try distance equals time cubed.







                share|cite|improve this answer














                share|cite|improve this answer



                share|cite|improve this answer








                edited 7 hours ago

























                answered 8 hours ago









                James MontagneJames Montagne

                114




                114











                • $begingroup$
                  This does not provide an answer to the question. Once you have sufficient reputation you will be able to comment on any post; instead, provide answers that don't require clarification from the asker. - From Review
                  $endgroup$
                  – Cosmas Zachos
                  2 hours ago
















                • $begingroup$
                  This does not provide an answer to the question. Once you have sufficient reputation you will be able to comment on any post; instead, provide answers that don't require clarification from the asker. - From Review
                  $endgroup$
                  – Cosmas Zachos
                  2 hours ago















                $begingroup$
                This does not provide an answer to the question. Once you have sufficient reputation you will be able to comment on any post; instead, provide answers that don't require clarification from the asker. - From Review
                $endgroup$
                – Cosmas Zachos
                2 hours ago




                $begingroup$
                This does not provide an answer to the question. Once you have sufficient reputation you will be able to comment on any post; instead, provide answers that don't require clarification from the asker. - From Review
                $endgroup$
                – Cosmas Zachos
                2 hours ago











                1












                $begingroup$

                One of the things that allowed us to invent and understand Newton's laws, which govern the motion of classical objects, is that they have the mathematical property of linearity. Linearity is a class of mathematical behaviors that people tend to be good at. If there are two or more forces acting on an object, the net force is the simple vector sum of the forces. If there is a system made of several masses, the total mass is the simple arithmetic sum of the individual masses.



                For example, an Aristotelian misconception that was properly refuted until the Renaissance is that "heavier objects fall faster." An argument against this, which I believe was due to Galileo, is a thought experiment, as follows. Imagine three identical cannonballs dropped from the same height: they should fall at the same rate because they all have the same mass. But tie two of them together with a low-mass connector, like glue or silk, and you have two objects whose masses are different by a factor of two. Can connecting two heavy masses with a silk thread change their fall time by a factor of two? It's a clever thought experiment.



                In your formulation, two identical masses $m$ accelerating in tandem with the same acceleration $a$ would have the same amount of force,



                $$
                F = m^2 a,
                $$



                acting on them. But if you tied them together so that you had a single object with mass $2m$ being acted upon by a force $2F$, then you would have



                $$
                2F = (2m)^2 a
                $$



                which predicts a different, smaller acceleration. Either you'd have to come up with an additional non-linear rule for adding forces, or you'd be describing a world that is different from the one where we live.






                share|cite|improve this answer









                $endgroup$

















                  1












                  $begingroup$

                  One of the things that allowed us to invent and understand Newton's laws, which govern the motion of classical objects, is that they have the mathematical property of linearity. Linearity is a class of mathematical behaviors that people tend to be good at. If there are two or more forces acting on an object, the net force is the simple vector sum of the forces. If there is a system made of several masses, the total mass is the simple arithmetic sum of the individual masses.



                  For example, an Aristotelian misconception that was properly refuted until the Renaissance is that "heavier objects fall faster." An argument against this, which I believe was due to Galileo, is a thought experiment, as follows. Imagine three identical cannonballs dropped from the same height: they should fall at the same rate because they all have the same mass. But tie two of them together with a low-mass connector, like glue or silk, and you have two objects whose masses are different by a factor of two. Can connecting two heavy masses with a silk thread change their fall time by a factor of two? It's a clever thought experiment.



                  In your formulation, two identical masses $m$ accelerating in tandem with the same acceleration $a$ would have the same amount of force,



                  $$
                  F = m^2 a,
                  $$



                  acting on them. But if you tied them together so that you had a single object with mass $2m$ being acted upon by a force $2F$, then you would have



                  $$
                  2F = (2m)^2 a
                  $$



                  which predicts a different, smaller acceleration. Either you'd have to come up with an additional non-linear rule for adding forces, or you'd be describing a world that is different from the one where we live.






                  share|cite|improve this answer









                  $endgroup$















                    1












                    1








                    1





                    $begingroup$

                    One of the things that allowed us to invent and understand Newton's laws, which govern the motion of classical objects, is that they have the mathematical property of linearity. Linearity is a class of mathematical behaviors that people tend to be good at. If there are two or more forces acting on an object, the net force is the simple vector sum of the forces. If there is a system made of several masses, the total mass is the simple arithmetic sum of the individual masses.



                    For example, an Aristotelian misconception that was properly refuted until the Renaissance is that "heavier objects fall faster." An argument against this, which I believe was due to Galileo, is a thought experiment, as follows. Imagine three identical cannonballs dropped from the same height: they should fall at the same rate because they all have the same mass. But tie two of them together with a low-mass connector, like glue or silk, and you have two objects whose masses are different by a factor of two. Can connecting two heavy masses with a silk thread change their fall time by a factor of two? It's a clever thought experiment.



                    In your formulation, two identical masses $m$ accelerating in tandem with the same acceleration $a$ would have the same amount of force,



                    $$
                    F = m^2 a,
                    $$



                    acting on them. But if you tied them together so that you had a single object with mass $2m$ being acted upon by a force $2F$, then you would have



                    $$
                    2F = (2m)^2 a
                    $$



                    which predicts a different, smaller acceleration. Either you'd have to come up with an additional non-linear rule for adding forces, or you'd be describing a world that is different from the one where we live.






                    share|cite|improve this answer









                    $endgroup$



                    One of the things that allowed us to invent and understand Newton's laws, which govern the motion of classical objects, is that they have the mathematical property of linearity. Linearity is a class of mathematical behaviors that people tend to be good at. If there are two or more forces acting on an object, the net force is the simple vector sum of the forces. If there is a system made of several masses, the total mass is the simple arithmetic sum of the individual masses.



                    For example, an Aristotelian misconception that was properly refuted until the Renaissance is that "heavier objects fall faster." An argument against this, which I believe was due to Galileo, is a thought experiment, as follows. Imagine three identical cannonballs dropped from the same height: they should fall at the same rate because they all have the same mass. But tie two of them together with a low-mass connector, like glue or silk, and you have two objects whose masses are different by a factor of two. Can connecting two heavy masses with a silk thread change their fall time by a factor of two? It's a clever thought experiment.



                    In your formulation, two identical masses $m$ accelerating in tandem with the same acceleration $a$ would have the same amount of force,



                    $$
                    F = m^2 a,
                    $$



                    acting on them. But if you tied them together so that you had a single object with mass $2m$ being acted upon by a force $2F$, then you would have



                    $$
                    2F = (2m)^2 a
                    $$



                    which predicts a different, smaller acceleration. Either you'd have to come up with an additional non-linear rule for adding forces, or you'd be describing a world that is different from the one where we live.







                    share|cite|improve this answer












                    share|cite|improve this answer



                    share|cite|improve this answer










                    answered 2 hours ago









                    robrob

                    41.7k1076173




                    41.7k1076173





















                        0












                        $begingroup$

                        No matter the transformations you make into Newton's second law, as long as it's reverse transformation admits $rm a$ to be isolated.



                        $F(r,v,t)$ in Newton's second law as expressed by equation
                        $$ F(r,v,t)=ma tag1$$



                        Is a very abstract concept that must be adapted to solve the observed motions in the universe. More importantly, $F(r,v,t)$ is an abstraction of some kind of position, time, and velocity function that satisfy the experimental observations. Since equation (1) can be written



                        $$F'(r,v,t)=a tag2$$



                        and $F'(r,v,t)$ has no form a priori, any transformation in (1) that can be reversed into (2), will give the same results as Newton's second law. However, What one would call force is not necessarily what we currently call it but they should be easily converted into one another.



                        For example, lets analyze the definition of gravitational force in the advent that Newton's second law were



                        $$F(x,v,t)=2ma^2 tag3$$



                        In this particular case, the definition of gravitational force $F_G$ here must satisfy (at least) the following observed facts:



                        1. (By Galilee): “The acceleration of a falling body doesn’t depend on its mass”;

                        2. (By Kepler): “The motion of planets around the sun are ellipses…”

                        The simplest definition of $F_G$ that satisfies both conditions is



                        $$F_G=frac4m^2r^4 tag4$$



                        However, with time, people would realize that $sqrtF_G$ was actually a better physical quantity, since more intuitive generalization of it, based on geometrical insights like Gauss law would arise. The same goes for the use of $m$ instead of $2m$.






                        share|cite|improve this answer











                        $endgroup$

















                          0












                          $begingroup$

                          No matter the transformations you make into Newton's second law, as long as it's reverse transformation admits $rm a$ to be isolated.



                          $F(r,v,t)$ in Newton's second law as expressed by equation
                          $$ F(r,v,t)=ma tag1$$



                          Is a very abstract concept that must be adapted to solve the observed motions in the universe. More importantly, $F(r,v,t)$ is an abstraction of some kind of position, time, and velocity function that satisfy the experimental observations. Since equation (1) can be written



                          $$F'(r,v,t)=a tag2$$



                          and $F'(r,v,t)$ has no form a priori, any transformation in (1) that can be reversed into (2), will give the same results as Newton's second law. However, What one would call force is not necessarily what we currently call it but they should be easily converted into one another.



                          For example, lets analyze the definition of gravitational force in the advent that Newton's second law were



                          $$F(x,v,t)=2ma^2 tag3$$



                          In this particular case, the definition of gravitational force $F_G$ here must satisfy (at least) the following observed facts:



                          1. (By Galilee): “The acceleration of a falling body doesn’t depend on its mass”;

                          2. (By Kepler): “The motion of planets around the sun are ellipses…”

                          The simplest definition of $F_G$ that satisfies both conditions is



                          $$F_G=frac4m^2r^4 tag4$$



                          However, with time, people would realize that $sqrtF_G$ was actually a better physical quantity, since more intuitive generalization of it, based on geometrical insights like Gauss law would arise. The same goes for the use of $m$ instead of $2m$.






                          share|cite|improve this answer











                          $endgroup$















                            0












                            0








                            0





                            $begingroup$

                            No matter the transformations you make into Newton's second law, as long as it's reverse transformation admits $rm a$ to be isolated.



                            $F(r,v,t)$ in Newton's second law as expressed by equation
                            $$ F(r,v,t)=ma tag1$$



                            Is a very abstract concept that must be adapted to solve the observed motions in the universe. More importantly, $F(r,v,t)$ is an abstraction of some kind of position, time, and velocity function that satisfy the experimental observations. Since equation (1) can be written



                            $$F'(r,v,t)=a tag2$$



                            and $F'(r,v,t)$ has no form a priori, any transformation in (1) that can be reversed into (2), will give the same results as Newton's second law. However, What one would call force is not necessarily what we currently call it but they should be easily converted into one another.



                            For example, lets analyze the definition of gravitational force in the advent that Newton's second law were



                            $$F(x,v,t)=2ma^2 tag3$$



                            In this particular case, the definition of gravitational force $F_G$ here must satisfy (at least) the following observed facts:



                            1. (By Galilee): “The acceleration of a falling body doesn’t depend on its mass”;

                            2. (By Kepler): “The motion of planets around the sun are ellipses…”

                            The simplest definition of $F_G$ that satisfies both conditions is



                            $$F_G=frac4m^2r^4 tag4$$



                            However, with time, people would realize that $sqrtF_G$ was actually a better physical quantity, since more intuitive generalization of it, based on geometrical insights like Gauss law would arise. The same goes for the use of $m$ instead of $2m$.






                            share|cite|improve this answer











                            $endgroup$



                            No matter the transformations you make into Newton's second law, as long as it's reverse transformation admits $rm a$ to be isolated.



                            $F(r,v,t)$ in Newton's second law as expressed by equation
                            $$ F(r,v,t)=ma tag1$$



                            Is a very abstract concept that must be adapted to solve the observed motions in the universe. More importantly, $F(r,v,t)$ is an abstraction of some kind of position, time, and velocity function that satisfy the experimental observations. Since equation (1) can be written



                            $$F'(r,v,t)=a tag2$$



                            and $F'(r,v,t)$ has no form a priori, any transformation in (1) that can be reversed into (2), will give the same results as Newton's second law. However, What one would call force is not necessarily what we currently call it but they should be easily converted into one another.



                            For example, lets analyze the definition of gravitational force in the advent that Newton's second law were



                            $$F(x,v,t)=2ma^2 tag3$$



                            In this particular case, the definition of gravitational force $F_G$ here must satisfy (at least) the following observed facts:



                            1. (By Galilee): “The acceleration of a falling body doesn’t depend on its mass”;

                            2. (By Kepler): “The motion of planets around the sun are ellipses…”

                            The simplest definition of $F_G$ that satisfies both conditions is



                            $$F_G=frac4m^2r^4 tag4$$



                            However, with time, people would realize that $sqrtF_G$ was actually a better physical quantity, since more intuitive generalization of it, based on geometrical insights like Gauss law would arise. The same goes for the use of $m$ instead of $2m$.







                            share|cite|improve this answer














                            share|cite|improve this answer



                            share|cite|improve this answer








                            edited 2 hours ago

























                            answered 3 hours ago









                            J. ManuelJ. Manuel

                            1,020222




                            1,020222





















                                0












                                $begingroup$

                                I have not read all the answers but most of them are conceptually wrong. Newton's second law does not state that force is proportional to acceleration. Mass is not just a proportionality constant.



                                Newton's second law of motion states that




                                The rate of change of motion(momentum) is directly proportional to the
                                motive force and the direction of the change is parallel to the force.




                                Hence instead of $Falpha a$ we have $vecFalpha fracdvecpdt$. Changing the proportionality into equality
                                $$vecF = kfracd(mvecv)dt$$
                                Solving through
                                $$vecF = kmfracdvecvdt + kvecvfracdmdt$$
                                In most cases mass of system is constant and hence the second term becomes zero, but the most general form of second law is as given above.



                                Now returning to the question. What if $k$ is something other than $1$. That will not change the physics. The only difference will be that accelerating objects will be harder (if $k>1$).



                                The other question is invalid because $m²$ is not possible since $vecp=mvecv$. One can again ask why is momentum not $m²vecv$? You can consider momentum as $m²vecv$ just don't call $m$ as the mass, since momentum is defined that way.






                                share|cite|improve this answer









                                $endgroup$












                                • $begingroup$
                                  Consider using propto in place of alpha.
                                  $endgroup$
                                  – rob
                                  13 mins ago















                                0












                                $begingroup$

                                I have not read all the answers but most of them are conceptually wrong. Newton's second law does not state that force is proportional to acceleration. Mass is not just a proportionality constant.



                                Newton's second law of motion states that




                                The rate of change of motion(momentum) is directly proportional to the
                                motive force and the direction of the change is parallel to the force.




                                Hence instead of $Falpha a$ we have $vecFalpha fracdvecpdt$. Changing the proportionality into equality
                                $$vecF = kfracd(mvecv)dt$$
                                Solving through
                                $$vecF = kmfracdvecvdt + kvecvfracdmdt$$
                                In most cases mass of system is constant and hence the second term becomes zero, but the most general form of second law is as given above.



                                Now returning to the question. What if $k$ is something other than $1$. That will not change the physics. The only difference will be that accelerating objects will be harder (if $k>1$).



                                The other question is invalid because $m²$ is not possible since $vecp=mvecv$. One can again ask why is momentum not $m²vecv$? You can consider momentum as $m²vecv$ just don't call $m$ as the mass, since momentum is defined that way.






                                share|cite|improve this answer









                                $endgroup$












                                • $begingroup$
                                  Consider using propto in place of alpha.
                                  $endgroup$
                                  – rob
                                  13 mins ago













                                0












                                0








                                0





                                $begingroup$

                                I have not read all the answers but most of them are conceptually wrong. Newton's second law does not state that force is proportional to acceleration. Mass is not just a proportionality constant.



                                Newton's second law of motion states that




                                The rate of change of motion(momentum) is directly proportional to the
                                motive force and the direction of the change is parallel to the force.




                                Hence instead of $Falpha a$ we have $vecFalpha fracdvecpdt$. Changing the proportionality into equality
                                $$vecF = kfracd(mvecv)dt$$
                                Solving through
                                $$vecF = kmfracdvecvdt + kvecvfracdmdt$$
                                In most cases mass of system is constant and hence the second term becomes zero, but the most general form of second law is as given above.



                                Now returning to the question. What if $k$ is something other than $1$. That will not change the physics. The only difference will be that accelerating objects will be harder (if $k>1$).



                                The other question is invalid because $m²$ is not possible since $vecp=mvecv$. One can again ask why is momentum not $m²vecv$? You can consider momentum as $m²vecv$ just don't call $m$ as the mass, since momentum is defined that way.






                                share|cite|improve this answer









                                $endgroup$



                                I have not read all the answers but most of them are conceptually wrong. Newton's second law does not state that force is proportional to acceleration. Mass is not just a proportionality constant.



                                Newton's second law of motion states that




                                The rate of change of motion(momentum) is directly proportional to the
                                motive force and the direction of the change is parallel to the force.




                                Hence instead of $Falpha a$ we have $vecFalpha fracdvecpdt$. Changing the proportionality into equality
                                $$vecF = kfracd(mvecv)dt$$
                                Solving through
                                $$vecF = kmfracdvecvdt + kvecvfracdmdt$$
                                In most cases mass of system is constant and hence the second term becomes zero, but the most general form of second law is as given above.



                                Now returning to the question. What if $k$ is something other than $1$. That will not change the physics. The only difference will be that accelerating objects will be harder (if $k>1$).



                                The other question is invalid because $m²$ is not possible since $vecp=mvecv$. One can again ask why is momentum not $m²vecv$? You can consider momentum as $m²vecv$ just don't call $m$ as the mass, since momentum is defined that way.







                                share|cite|improve this answer












                                share|cite|improve this answer



                                share|cite|improve this answer










                                answered 1 hour ago









                                Manvendra SomvanshiManvendra Somvanshi

                                31919




                                31919











                                • $begingroup$
                                  Consider using propto in place of alpha.
                                  $endgroup$
                                  – rob
                                  13 mins ago
















                                • $begingroup$
                                  Consider using propto in place of alpha.
                                  $endgroup$
                                  – rob
                                  13 mins ago















                                $begingroup$
                                Consider using propto in place of alpha.
                                $endgroup$
                                – rob
                                13 mins ago




                                $begingroup$
                                Consider using propto in place of alpha.
                                $endgroup$
                                – rob
                                13 mins ago










                                pooja paliwal is a new contributor. Be nice, and check out our Code of Conduct.









                                draft saved

                                draft discarded


















                                pooja paliwal is a new contributor. Be nice, and check out our Code of Conduct.












                                pooja paliwal is a new contributor. Be nice, and check out our Code of Conduct.











                                pooja paliwal is a new contributor. Be nice, and check out our Code of Conduct.














                                Thanks for contributing an answer to Physics Stack Exchange!


                                • Please be sure to answer the question. Provide details and share your research!

                                But avoid


                                • Asking for help, clarification, or responding to other answers.

                                • Making statements based on opinion; back them up with references or personal experience.

                                Use MathJax to format equations. MathJax reference.


                                To learn more, see our tips on writing great answers.




                                draft saved


                                draft discarded














                                StackExchange.ready(
                                function ()
                                StackExchange.openid.initPostLogin('.new-post-login', 'https%3a%2f%2fphysics.stackexchange.com%2fquestions%2f475733%2fwhat-if-force-was-not-mass-times-acceleration%23new-answer', 'question_page');

                                );

                                Post as a guest















                                Required, but never shown





















































                                Required, but never shown














                                Required, but never shown












                                Required, but never shown







                                Required, but never shown

































                                Required, but never shown














                                Required, but never shown












                                Required, but never shown







                                Required, but never shown







                                Popular posts from this blog

                                How to create a command for the “strange m” symbol in latex? Announcing the arrival of Valued Associate #679: Cesar Manara Planned maintenance scheduled April 23, 2019 at 23:30 UTC (7:30pm US/Eastern)How do you make your own symbol when Detexify fails?Writing bold small caps with mathpazo packageplus-minus symbol with parenthesis around the minus signGreek character in Beamer document titleHow to create dashed right arrow over symbol?Currency symbol: Turkish LiraDouble prec as a single symbol?Plus Sign Too Big; How to Call adfbullet?Is there a TeX macro for three-legged pi?How do I get my integral-like symbol to align like the integral?How to selectively substitute a letter with another symbol representing the same letterHow do I generate a less than symbol and vertical bar that are the same height?

                                Българска екзархия Съдържание История | Български екзарси | Вижте също | Външни препратки | Литература | Бележки | НавигацияУстав за управлението на българската екзархия. Цариград, 1870Слово на Ловешкия митрополит Иларион при откриването на Българския народен събор в Цариград на 23. II. 1870 г.Българската правда и гръцката кривда. От С. М. (= Софийски Мелетий). Цариград, 1872Предстоятели на Българската екзархияПодмененият ВеликденИнформационна агенция „Фокус“Димитър Ризов. Българите в техните исторически, етнографически и политически граници (Атлас съдържащ 40 карти). Berlin, Königliche Hoflithographie, Hof-Buch- und -Steindruckerei Wilhelm Greve, 1917Report of the International Commission to Inquire into the Causes and Conduct of the Balkan Wars

                                Category:Tremithousa Media in category "Tremithousa"Navigation menuUpload media34° 49′ 02.7″ N, 32° 26′ 37.32″ EOpenStreetMapGoogle EarthProximityramaReasonatorScholiaStatisticsWikiShootMe